/ Forside / Karriere / Uddannelse / Højere uddannelser / Nyhedsindlæg
Login
Glemt dit kodeord?
Brugernavn

Kodeord


Reklame
Top 10 brugere
Højere uddannelser
#NavnPoint
Nordsted1 1588
erling_l 1224
ans 1150
dova 895
gert_h 800
molokyle 661
berpox 610
creamygirl 610
3773 570
10  jomfruane 570
Invertible matricer over F_p
Fra : Rasmus Villemoes


Dato : 09-02-04 22:07

Hvis man ser på en "tilfældig" n×n-matrix A med reelle (komplekse,
rationale...) indgange, er det forholdsvis indlysende, at
sandsynligheden for at A er invertibel må være 1 [i hvert fald for
alle fornuftige definitioner af "tilfældig matrix"; omend jeg ikke
lige på stående fod kan finde på en konkret, god definition].

Hvis man nu i stedet kigger på matricer over legemet F_p (altså
legemet med p elementer, p et primtal) er der lige pludselig kun
endeligt mange n×n-matricer (faktisk præcis p^{n^2}), og da "en del"
af disse er singulære, er sandsynligheden P(n, p) for at en tilfældig
matrix i Mat_n(F_p) er invertibel strengt mindre end 1. Men findes der
en formel for beregning af P(n, p)? Hvad hvis man erstatter p med p^k?
Jeg behøver ikke et eksplicit funktionsudtryk, men jeg er interesseret
i den asymptotiske opførsel når n og/eller p [p^k] bliver stor.

Det er naturligvis muligt at beregne konkrete sandsynligheder ved
brute force, men det at udregne determinanterne af (p^k)^{n^2}
matricer er nok ikke den smarteste metode der findes...

Mvh Rasmus

--

 
 
Henning Makholm (09-02-2004)
Kommentar
Fra : Henning Makholm


Dato : 09-02-04 23:51

Scripsit Rasmus Villemoes <burner+usenet@imf.au.dk>

> Hvis man nu i stedet kigger på matricer over legemet F_p (altså
> legemet med p elementer, p et primtal) er der lige pludselig kun
> endeligt mange n×n-matricer (faktisk præcis p^{n^2}), og da "en del"
> af disse er singulære, er sandsynligheden P(n, p) for at en tilfældig
> matrix i Mat_n(F_p) er invertibel strengt mindre end 1.

Hvad med følgende:

Vi vælger matricen ved en søjle ad gangen at smide tilfældigt valgte
(og ligefordelte) elementer ind på indgangene. Så snart vi véd at
søjlerne ikke er lineært uafhængige stopper vi.

I første søjle mister vi med det samme invertibilitet hvis og kun hvis
søjlen er 0. Det vil sige at sandsynligheden for at overleve første
søjle er 1-(1/p^n) = 1-(1/p)^(-n).

Hvis vi overlever første søjle, er sandsynligheden for at tabe i anden
søjle p/p^n (idet vektorrummet udspændt af den første søjle har
netop p elementer). Sandsynligheden for at *overleve* anden søjle er
derfor 1-(p/p^n) = 1-(1/p)^(n-1).

Tilsvarende er sandsynligheden for at overleve tredje søjle 1-(1/p)^(n-2).

Sandsynligheden for at finde en hel invertibel matrix bliver

P(Inv) = (1-(1/p)^n)*(1-(1/p)^(n-2))*...*(1-(1/p)^1)

> Jeg behøver ikke et eksplicit funktionsudtryk, men jeg er interesseret
> i den asymptotiske opførsel når n og/eller p [p^k] bliver stor.

Udtrykket ovenfor er et polynomium i 1/p, så når p bliver stor kan vi
vel nøjes med leddene af lavest grad, og dem kan vi forholdsvis nemt
finde ved håndkraft. Her er indtil tiende grad, (hvis altså n>10):

P(Inv) = 1 - (1/p) - (1/p)² + (1/p)^5 + (1/p)^7 + ...

Koefficientfølgen er i almindelighed Sloane's A010815. [1] Sloane
giver en lukket formel for koefficienter af grad op til n:

a(n) = (-1)^m if n is of the form m(3m+-1)/2; otherwise a(n)=0.

Idet alle koefficienter her er numerisk højst 1, kan vi nok gå ud fra
at P(Inv) er forholdsvis upåvirket hvis vi lader n vokse sig stor.

[1] http://www.research.att.com/projects/OEIS?Anum=A010815

> Hvad hvis man erstatter p med p^k?

Ovenstående forudsætter kun at legemet er endeligt.

--
Henning Makholm "... not one has been remembered from the time
when the author studied freshman physics. Quite the
contrary: he merely remembers that such and such is true, and to
explain it he invents a demonstration at the moment it is needed."

Jeppe Stig Nielsen (10-02-2004)
Kommentar
Fra : Jeppe Stig Nielsen


Dato : 10-02-04 11:51

Henning Makholm wrote:
>
> Scripsit Rasmus Villemoes <burner+usenet@imf.au.dk>
>
> > Hvis man nu i stedet kigger på matricer over legemet F_p (altså
> > legemet med p elementer, p et primtal) er der lige pludselig kun
> > endeligt mange n×n-matricer (faktisk præcis p^{n^2}), og da "en del"
> > af disse er singulære, er sandsynligheden P(n, p) for at en tilfældig
> > matrix i Mat_n(F_p) er invertibel strengt mindre end 1.
>
> Hvad med følgende:

Ja, det holder!

>
> Sandsynligheden for at finde en hel invertibel matrix bliver
>
> P(Inv) = (1-(1/p)^n)*(1-(1/p)^(n-2))*...*(1-(1/p)^1)

Ja, det er rigtigt (bortset fra at du kommer til at springe en faktor
over). Og som du skriver holder det stadig hvis legemet har L=p^k
elementer. Antallet af invertible n×n-matricer er da

produkt{fra j=0, til n-1: (L^n - L^j) }

med samme argument som det du bruger. Nemlig at antallet af måder at
vælge den (j+1)'te søjle på er antallet af vektorer i alt, L^n, minus
antallet L^j af vektorer der ligger i det »forbudte« underrum udspændt
af de j allerede valgte søjler.

Jeg udregner for L=2 følgen: 1,6,168,20160,... Denne findes hos Sloane:
http://www.research.att.com/projects/OEIS?Anum=A002884


--
Jeppe Stig Nielsen <URL:http://jeppesn.dk/>. «

"Je n'ai pas eu besoin de cette hypothèse (I had no need of that
hypothesis)" --- Laplace (1749-1827)

Jeppe Stig Nielsen (09-02-2004)
Kommentar
Fra : Jeppe Stig Nielsen


Dato : 09-02-04 23:56

Rasmus Villemoes wrote:
>
> Det er naturligvis muligt at beregne konkrete sandsynligheder ved
> brute force, men det at udregne determinanterne af (p^k)^{n^2}
> matricer er nok ikke den smarteste metode der findes...

Har du kigget i Sloane? http://www.research.att.com/~njas/sequences/

Hvis du fx kan beregne antallet i specialtilfældet p^k=2^1 med brute
force, kan det være følgen findes hos Sloane.

--
Jeppe Stig Nielsen <URL:http://jeppesn.dk/>. «

"Je n'ai pas eu besoin de cette hypothèse (I had no need of that
hypothesis)" --- Laplace (1749-1827)

Henning Makholm (10-02-2004)
Kommentar
Fra : Henning Makholm


Dato : 10-02-04 00:07

Scripsit Rasmus Villemoes <burner+usenet@imf.au.dk>

> Hvis man ser på en "tilfældig" n×n-matrix A med reelle (komplekse,
> rationale...) indgange, er det forholdsvis indlysende, at
> sandsynligheden for at A er invertibel må være 1

Stryg "rationale", hvis n>1:

> [i hvert fald for alle fornuftige definitioner af "tilfældig
> matrix";

En fornuftig definition bør indebære at elementerne i matricen er
uafhængige stokastiske variable med samme sandsynlighedsfordeling.

Ved en hvilken som helst sandsynlighedsfordeling med tælleligt
udfaldsrum er der (så vidt jeg husker) mindst ét udfald som har en
positiv sandsynlighed for at forekomme. Kald dette udfald x, og dets
sandsynlighed q>0.

Den tilfældige matrix kan ikke være invertibel hvis alle de 2n
elementer i de to højreste søjler er x. Sandsynligheden for at dette
sker er q^2n > 0. Derfor kan sandsynligheden for en invertibel matrix
med rationale elementer ikke være 1.

--
Henning Makholm "I always thought being *real* sad
would be *cooler* than acting *fake*
sad, but it's not. It's not cool at *all*."

Stefan Holm (10-02-2004)
Kommentar
Fra : Stefan Holm


Dato : 10-02-04 00:14

Henning Makholm <henning@makholm.net> writes:

> Ved en hvilken som helst sandsynlighedsfordeling med tælleligt
> udfaldsrum er der (så vidt jeg husker) mindst ét udfald som har en
> positiv sandsynlighed for at forekomme.

Mål er tælleligt additive, så hvis alle punkter har sandsynlighed 0,
vil den samlede sandsynlighedsmasse være 0.

--
Stefan Holm
"Hmm... their construction should be exceedingly simple. I think."

Niels L. Ellegaard (10-02-2004)
Kommentar
Fra : Niels L. Ellegaard


Dato : 10-02-04 09:16

Rasmus Villemoes <burner+usenet@imf.au.dk> writes:

> Hvis man nu i stedet kigger på matricer over legemet F_p (altså
> legemet med p elementer, p et primtal) er der lige pludselig kun
> endeligt mange n×n-matricer (faktisk præcis p^{n^2}), og da "en del"
> af disse er singulære, er sandsynligheden P(n, p) for at en
> tilfældig matrix i Mat_n(F_p) er invertibel strengt mindre end
> 1. Men findes der en formel for beregning af P(n, p)? Hvad hvis man
> erstatter p med p^k? Jeg behøver ikke et eksplicit funktionsudtryk,
> men jeg er interesseret i den asymptotiske opførsel når n og/eller p
> [p^k] bliver stor.

Jeg har ikke et svar på dit spørgsmål, men her er en artikel du måske
kan bruge til at starte med at søge videre efter dit svar

Kent Morrison: Eigenvalues of matrices over finite fields
http://www.calpoly.edu/~kmorriso/Research/ERMFF.pdf

Så vidt jeg kan se undersøger de grænsen n -> uendelig, men jeg må med
skam erkende at jeg ikke har regnet det hele efter.

--
Niels L Ellegaard http://dirac.ruc.dk/~gnalle/

Søg
Reklame
Statistik
Spørgsmål : 177501
Tips : 31968
Nyheder : 719565
Indlæg : 6408527
Brugere : 218887

Månedens bedste
Årets bedste
Sidste års bedste